LSAT and Law School Admissions Forum

Get expert LSAT preparation and law school admissions advice from PowerScore Test Preparation.

 Administrator
PowerScore Staff
  • PowerScore Staff
  • Posts: 8916
  • Joined: Feb 02, 2011
|
#22790
Complete Question Explanation

Weaken—CE. The correct answer choice is (A)

This stimulus cites a causal hypothesis advanced by researchers after a recent study: among several hundred female physicians, the more folate and B6 each participant consumed, the less likely she was to develop coronary disease. Based on these results, researchers concluded that folate and B6 vitamins help to block heart disease in women.

This is a causal conclusion, followed by a weaken question. If an answer provides other possible causes of the study results, that choice will likely be the correct answer.

Answer choice (A): This is the correct answer choice.
The study showed a lower rate of coronary disease among those who consumed folate and B6, but if those same foods contained other ingredients known to inhibit heart disease, then that provides a viable alternative cause of the research results.

Answer choice (B): The stimulus makes no reference to men, so this choice would not refute the causal argument in the stimulus, nor weaken any part of the argument.

Answer choice (C): Knowledge of the link between diet and health would not affect the strength of the argument in the stimulus, because all of the participants were doctors, creating a “level playing field” with regard to this knowledge (that is, all of the participants were exposed to this same information).

Answer choice (D): Like incorrect answer choice (C) above, this choice makes no distinction discussed in the stimulus; none of the physicians were screened in advance, so this choice does not distinguish between the participants who consumed more folate and B6 and those who consumed less, or those who were more likely to develop coronary disease and those who were less likely.

Answer choice (E): Regardless of the quantities of folate and B6 present in most foods, the results of the study reflected a correlation, based upon which the researchers drew a causal conclusion. This choice does not weaken the causal argument from the stimulus, so this answer choice is incorrect.
 ellenb
  • Posts: 260
  • Joined: Oct 22, 2012
|
#12301
Dear Powerscore,

I got this question right, however, I want to make sure I know why answer choice D is wrong.
In a way we needed to make sure that we compare apples to apples and not apples to oranges, is it because here it is only one group and there is not mentioning of prescrened and not pre-screned doctors?

Thanks in advance!

Ellen
 Adam Tyson
PowerScore Staff
  • PowerScore Staff
  • Posts: 5153
  • Joined: Apr 14, 2011
|
#12303
I think that's a good way to look at it, Ellen - screening just isn't an issue here. The study revealed a correlation between heart disease and the intake of certain vitamins, and there was no need to differentiate between those who had been prescreened and those who had not. Screening was just a distraction here.

The key was to suggest our favorite weaken attack for causal reasoning, an alternate cause. Answer A, as you figured out, gives us that alternate cause (the other, mysterious nutrients that just happen to also go hand in hand with those vitamins).

Good job!
 dhilliard
  • Posts: 2
  • Joined: Aug 28, 2014
|
#16235
Can anyone please help me by telling me how they would approach #9 December 1995 LR2?
Thanks!
 Emily Haney-Caron
PowerScore Staff
  • PowerScore Staff
  • Posts: 577
  • Joined: Jan 12, 2012
|
#16241
Hi dhilliard,

As soon as I read this stimulus, I started thinking that what would really weaken the argument is something showing an alternative possible cause of coronary disease that happened to coexist with lower folate and B6 consumption. Basically, I'm looking for something that shows that just because there is a correlation between coronary disease and low folate/B6, that doesn't mean one caused the other. That's what we find with answer A, which establishes that a lack of non-vitamin nutrients could be the cause of the coronary disease, and the lack of folate and B6 could just be a coincidence.
 jessicamorehead
  • Posts: 84
  • Joined: Jul 07, 2017
|
#37387
I got the correct answer, but I want to ensure that I understand why D was wrong.

I said D was wrong because it discussed preexisting heart conditions, which is not discussed in the author's argument/conclusion. The argument/conclusion discusses the DEVELOPMENT of heart disease in women, so talking about people who already have the heart disease is not permitted. Is my logic okay there?
User avatar
 Jonathan Evans
PowerScore Staff
  • PowerScore Staff
  • Posts: 726
  • Joined: Jun 09, 2016
|
#37510
Hi, Jessica,

Your point is certainly valid; it's true that the study focused on the "development" of heart disease rather than perhaps the recurrence of existing heart disease. However, just to play devil's advocate: a recurrence in preexisting heart disease could be misinterpreted as a development of new heart disease, especially if such conditions had not been screened for.

Nonetheless, there are a couple even more serious issues with answer choice (D) that prevent it from being the answer that most weakens the conclusion.

First, while the statement in (D) might undermine the overall integrity of the study used as evidence for the conclusion, this lack of screening in no way differentiates between women with higher B-vitamin intake and those with lower B-vitamin intake. Thus, the effect of this lack of screening might be expected to occur "across the board" and not have a significant effect on the relative results of the two groups.

Second, this hypothetical methodological issue is not germane to the main issues with the reasoning in this argument, that is, we are dealing primarily with causality here (does higher B-vitamin intake reduce incidence of heart disease?) rather than survey or sample problems. This observation is not intended to mean that the credited response to a weaken or strengthen question will never involve some secondary, minor flaw; however, if you are to choose an answer that either exploits or remedies a minor flaw, be sure that there is no other answer that doesn't do a better job exploiting or remedying the more salient, major flaw.

I hope this helps!
 lunsandy
  • Posts: 61
  • Joined: Oct 14, 2017
|
#40881
Hi Powerscore Staff!

I left A as a contender and ended up choosing C, which is the wrong answer. I end up choosing C because I was thinking "oh , of course physicians know more about health and diet than nonphysicans," and this weakens the conclusion because a a group of female physicians is surveyed to make a conclusion about the health of women in general. But physicians are suppose to make recommendations and there are suppose to be a difference in knowledge from physicians vs. non-physicians. That is wrong because we are not comparing the knowledge of the two groups in the conclusions. Like what the other Powerscore instructor mentioned, the physicians are in the same level of playing field. Physicians knowing more than nonphysicians does not weaken the conclusion about the folate and BF inhibiting the development of HD in women. Answer A introduces the idea of nonvitamin nutrients, which allows for the effect to happen (inhibit HD) without the cause (folate and B6). Is this the correct thought process?
 Francis O'Rourke
PowerScore Staff
  • PowerScore Staff
  • Posts: 471
  • Joined: Mar 10, 2017
|
#40991
Hi Lunsandy,
Answer A introduces the idea of nonvitamin nutrients, which allows for the effect to happen (inhibit HD) without the cause (folate and B6). Is this the correct thought process?
This is a slight misinterpretation. Answer choice (A) only provides a possible alternate cause; it does not claim that the reduced risk of coronary disease occurs without the presence of folate and B6.

According to this answer choice, the vitamins folate and B6 may still be present in the diets of these doctors. What it does say however is that other nutrients that are in foods with lots of folate and B6 may be the real cause. In this way, folate and B6 may still be correlated with reduced coronary disease, but they do not cause the reduction.

For example, imagine I argued that niacin helps the body produce strong, healthy bones, because people who drink milk develop strong bones and there is a lot of niacin in milk. You might object to me by saying milk also contains calcium which we know causes healthy bone growth. Your objection would not necessarily imply that the effect (strong bones) occurs without the supposed cause (niacin), but that there is an alternative cause which casts doubt on niacin's supposed role in bone development.
 ShannonOh22
  • Posts: 70
  • Joined: Aug 15, 2019
|
#67877
Hi powerscore - thought I would share the mistake I made on this question, in the event it may help others when deducing the correct answer. I chose D, which I have subsequently found out is completely irrelevant to the argument, but I realized I skipped right over A because it discussed food. I made the incorrect assumption that the stimulus was referring to a "DAILY" supplement with the two vitamins, instead of "DIETARY intake". This one had me stumped for a while, and i was all prepared to submit a novel-esque question to you guys demanding answers on A being correct, until I noticed my mistake was one of not reading closely enough. Hope this helps someone else avoid the frustration I just put myself through! :-)

Get the most out of your LSAT Prep Plus subscription.

Analyze and track your performance with our Testing and Analytics Package.